Select the correct answer from each drop-down menu.
The function f is given by the table of values as shown below.

x 1 2 3 4 5
f(x) 13 19 37 91 253
Use the given table to complete the statements.

The parent function of the function represented in the table is
.

If function f was translated down 4 units, the
-values would be
.

A point in the table for the transformed function would be
.

Answers

Answer 1

Answer:

3^x9, 15, 33, 87, 249(4, 87) for example

Step-by-step explanation:

a) First differences of the f(x) values in the table are ...

  19 -13 = 6, 37 -19 = 18, 91 -37 = 54, 253 -91 = 162

The second differences are not constant:

  18 -6 = 12, 54 -18 = 36, 162 -54 = 108

But, we notice that both the first and second differences have a common ratio. This is characteristic of an exponential function. The common ratio is 18/6 = 3, so the parent function is 3^x.

__

b) Translating a function down 4 units subtracts 4 from each y-value. The values of f(x) in the table would be ...

  9, 15, 33, 87, 249

__

c) The x-values of the function stay the same for a vertical translation, so the points in the table of the transformed function are ...

  (x, f(x)) = (1, 9), (2, 15), (3, 33), (4, 87), (5, 249)

Answer 2

Answer: I think this is it:

The parent function of the function represented in the table is exponential. If function f was translated down 4 units, the f(x)-values would be decreased by 4. A point in the table for the transformed function would be (4,87)

Step-by-step explanation: I got it right on Edmentum!


Related Questions

how to solve 8(y-7) in digits

Answers

Answer:

y = 7

Step-by-step explanation:

Equate the equation to equal 0.

8(y-7) = 0

Open up the bracket:

8y - 56 = 0

Add 56 to both sides:

8y = 56

Divide both sides by 8:

y = 7

Avanety of two types of snack packs are delivered to a store. The box plots compare the number of calories in each
snack pack of crackers to the number of calories in each snack pack of trail mix.
Number of Calories in Each Snack Pack
Crackers
Trail Mix
65
70
75
80
85
90
95
100 105 110 115
Which statement is true about the box plots?
The interquartile range of the trail mix data is greater than the range of the cracker data.
The value 70 is an outlier in the trail mix data
The upper quartile of the trail mix data is equal to the maximum value of the cracker data
O The number of calories in the packs of trail mix have a greater variation than the number of calories in the packs
of crackers

Answers

Answer:

The number of calories in the packs of trail mix have a greater variation than the number of calories in the packs

of crackers

Step-by-step explanation:

IQR of trail mix data = 105 - 90 = 15

The range of cracker data = 100 - 70 = 30.

Therefore, the first option is NOT TRUE.

To check if option 2 is correct, calculate the lower limit to see if 70 is below the lower limit. If 70 is below the lower limit, then it is an outlier in the trail mix data.

Thus, Lower Limit = [tex]Q_1 - 1.5(IQR)[/tex]

Q1 = 90,

IQR = 105 - 90 = 15

Lower Limit = [tex]90 - 1.5(15)[/tex]

Lower Limit = [tex]90 - 22.5 = 67.5[/tex]

70 is not less than the lower limit, therefore, 70 is not an outlier for the trail mix data. The second option is NOT TRUE.

The upper quartile of the trail mix data = 105.

The maximum value of the cracker data = 100.

Therefore, the third option is NOT TRUE.

Range can be used to determine how much variable there is in a data represented on a box plot. The greater the range value, the greater the variation.

Range of trail mix data = 115 - 70 = 45

Range of cracker data = 100 - 70 = 30.

The range value for the number of calories in trail mix is greater than that for cracker, therefore, the number of calories in the packs of trail mix have a greater variation than the number of calories in the packs

of crackers.

The fourth option is TRUE.

Answer: D. The number of calories in the packs of trail mix have a greater variation than the number of calories in the packs of crackers.

use the product of powers property to simplify the numeric expression.

4 1/3 • 4 1/5 = _____

Answers

Answer:

The value of [tex]4^{\dfrac{1}{3}} {\cdot} 4^{\dfrac{1}{5}[/tex]  is  [tex]4^{\dfrac{8}{15}}[/tex] .

Step-by-step explanation:

We need to simplify the numeric expression using property. The expression is as follows :

[tex]4^{\dfrac{1}{3}} {\cdot} 4^{\dfrac{1}{5}[/tex]

The property to be used is : [tex]x^a{\cdot} x^b=x^{a+b}[/tex]

This property is valid if the base is same. Here, base is x.

In this given problem, x = 4, a = 1/3 and b = 1/5

So,

[tex]4^{\dfrac{1}{3}} {\cdot} 4^{\dfrac{1}{5}}=4^{\dfrac{1}{3}+\dfrac{1}{5}}\\\\=4^{\dfrac{5+3}{15}}\\\\=4^{\dfrac{8}{15}}[/tex]

So, the value of [tex]4^{\dfrac{1}{3}} {\cdot} 4^{\dfrac{1}{5}[/tex]  is  [tex]4^{\dfrac{8}{15}}[/tex] .

I don't know how to do this ? can someone help me please​

Answers

3x-4+6x+2=5x+223x+6x-5x=22+4-29x-5x=26-24x=24 or = 4x-24x=24÷4x=6

please mark this answer as brainlist

If (4x-5) :(9x-5) = 3:8 find the value of x.​

Answers

Answer:

x is 5

Step-by-step explanation:

[tex] \frac{4x - 5}{9x - 5} = \frac{3}{8} \\ \\ 8(4x - 5) = 3(9x - 5) \\ 32x - 40 = 27x - 15 \\ 5x = 25 \\ x = \frac{25}{5} \\ \\ x = 5[/tex]

Step-by-step explanation:

as you can see as i solved above. all you need to do was to rationalize the both equations

Many stores run "secret sales": Shoppers receive cards that determine how large a discount they get, but the percentage is revealed by scratching off that black stuff only after the purchase has been totaled at the cash register. The store is required to reveal (in the fine print) the distribution of discounts available. Determine whether the following probability assignment is legitimate?


10% off 20% off 30% off 50% off
a. 0.2 0.2 0.2 0.2
b. 0.5 0.3 0.2 0.1
c. 0.8 0.1 0.05 0.05
d. 0.75 0.25 0.25 -0.25
e. 1 0 0 0

Answers

Answer:

b

Step-by-step explanation:

it makes the most senses the lower the discount the higher the chance

An urn has 6 white balls and 4 orange balls if Sam chooses 5 balls at random from the urn what is the probability that he chooses 2 white balls and 3 orange balls

Answers

Answer:

probability of picking white is 6/10

and probability of picking orange is 4/10

Step-by-step explanation:

so probability of picking 2 white and 3 orange will be 9/25 + 64/100 = 8/125

A translation T maps point B(-2,4) onto point B (3,-1). What is the translation T?
HELP PLSS!!!!

A- (x+5, y-5)
B- (x+5, y+5)
C- (x-5, y+5)
D- (x-5, y-5)

Answers

Answer:

(x+5, y-5) is the correct answer

Step-by-step explanation:

first, take -2 and 3 on x value; when you jump from -2 to 3, your answer is positive 5

second, take 4 and -1 on y value; when you jump from 4 to -1, your answer is negative 5

hence your answer for this question is a- (x+5, y-5)

Question
The point (-2,r) lies on the graph of 2x + y = 7 in the xy-plane. What is the value of r?

Answers

Answer: r = 11

Step-by-step explanation:

We know that the point (-2, r) lies on the graph of:

2*x + y = 7.

Then, if we that point is on the graph of the equation, we can replace the values and we will have:

2*(-2) + r = 7

and now we solve this for r-

-4 + r = 7

r = 7 + 4 = 11

r = 11

please help brainliest to correct answer

Answers

Answer:

Question to number 6 is-3

Question to number 7 is 3

Question to number 8 is 2 to the second power

Step-by-step explanation:

please correct me if I’m wrong and for number 8 I am correct it’s just I didn’t know how to put the little 2 on top of the big one

Step-by-step explanation:

question 6 is - 3

question 7 is 3

question 8 is 4

What are the divisible number(s) for 430?

Answers

Answer:

The numbers that 430 is divisible by are 1, 2, 5, 10, 43, 86, 215, and 430.

This is how you do it

The graph of F(x), shown below in pink, has the same shape as the graph of
G(x) = x3, shown in gray. Which of the following is the equation for F(x)?

Answers

Greetings from Brasil...

In this problem we have 2 translations: 4 units horizontal to the left and 3 units vertical to the bottom.

The translations are established as follows:

→ Horizontal

F(X + k) ⇒ k units to the left

F(X - k) ⇒ k units to the right

→ Vertical

F(X) + k ⇒ k units up

F(X) - k ⇒ k units down

In our problem, the function shifted 4 units horizontal to the left and 3 units vertical to the bottom.

F(X) = X³

4 units horizontal to the left: F(X + 4)

3 units vertical to the bottom: F(X + 4) - 3

So,

F(X) = X³

F(X + 4) - 3 = (X + 4)³ - 3

The transformed function is f ( x ) = ( x + 4 )³ - 3 and the graph is plotted

What happens when a function is transformed?

Every modification may be a part of a function's transformation.

Typically, they can be stretched (by multiplying outputs or inputs) or moved horizontally (by converting inputs) or vertically (by altering output).

If the horizontal axis is the input axis and the vertical is for outputs, if the initial function is y = f(x), then:

Vertical shift, often known as phase shift:

Y=f(x+c) with a left shift of c units (same output, but c units earlier)

Y=f(x-c) with a right shift of c units (same output, but c units late)

Vertical movement:

Y = f(x) + d units higher, up

Y = f(x) - d units lower, d

Stretching:

Stretching vertically by a factor of k: y = k f (x)

Stretching horizontally by a factor of k: y = f(x/k)

Given data ,

Let the function be represented as g ( x )

Now , the value of g ( x ) = x³

And , the transformed function has coordinates as A ( -4 , -3 )

So , when function is shifted 4 units to the left , we get

g' ( x ) = ( x + 4 )³

And , when the function is shifted vertically by 3 units down , we get

f ( x ) = ( x + 4 )³ - 3

Hence , the transformed function is f ( x ) = ( x + 4 )³ - 3

To learn more about transformation of functions click :

https://brainly.com/question/26896273

#SPJ7

Factorise the following using the Difference of Two Squares or Perfect Squares rule: a) (2x-2)^2 - (x+4)^2 b) (3x+4) (3x-4)

Answers

Answer:

Step-by-step explanation:

Hello, please consider the following.

a)

[tex](2x-2)^2 - (x+4)^2 \\\\=(2x-2-(x+4))(2x-2+x+4)\\\\=(2x-2-x-4)(3x+2)\\\\=\boxed{(x-6)(3x+2)}[/tex]

b)

[tex](3x+4) (3x-4)\\\\=(3x)^2-4^2\\\\=\boxed{9x^2-16}[/tex]

Thank you.

If f(x)=ax^2+bx+c and f(0)=-4 and f(1)=-2 and f(2)=6, what is the value of A and B and C?

Answers

Hello There!!

I'm not a 100% sure this right.

Step-by-step explanation:

f(x)=ax2+bx+c for which f(1)=0, f(-2)=6 and f(2)=-14

0 = a +b +c

6 =4a -2b +c

-14=4a+2b +c

subtract third equation from second to get

20 = -4b and so b=-5

first equation is now 5 = a+c

second is now -4=4a+c

subtract to get -9=3a and so a=-3

equation one now is 0=-3-5+c or c=8 Hope This Helps!!

2. An economist reports that 576 out of a sample of 1,200 middle-income American households participate in the stock market. A confidence interval of [0.468, 0.492] was calculated. What confidence level was used in this calculation

Answers

Answer:

Confidence level  = 59.46%

Step-by-step explanation:

Given that:

An economist reports that 576 out of a sample of 1,200 middle-income American households participate in the stock market.

sample mean = 576

sample size = 1200

The sample proportion [tex]\hat p[/tex] = x/n

The sample proportion [tex]\hat p[/tex] = 576/1200 = 0.48

A confidence interval of [0.468, 0.492] was calculated. What confidence level was used in this calculation?

The  confidence interval level can be determined by using the formula:

[tex]M.E =Z_{critical} \times \sqrt{\dfrac{\hat p (1- \hat p)}{n}}[/tex]

If the calculated confidence interval was [0.468, 0.492]

Then,

[tex]\hat p[/tex]  - M.E = 0.468

0.48 -M.E = 0.468

0.48 - 0.468 = M.E

0.012 = M.E

M.E = 0.012

NOW;

[tex]0. 012 =Z_{critical} \times \sqrt{\dfrac{0.48 (1- 0.48)}{1200}}[/tex]

[tex]0. 012 =Z_{critical} \times \sqrt{\dfrac{0.48 (0.52)}{1200}}[/tex]

[tex]0. 012 =Z_{critical} \times \sqrt{\dfrac{0.2496}{1200}}[/tex]

[tex]0. 012 =Z_{critical} \times \sqrt{2.08\times10^{-4}}[/tex]

[tex]0. 012 =Z_{critical} \times 0.01442[/tex]

[tex]\dfrac{0. 012}{0.01442} =Z_{critical}[/tex]

[tex]Z_{critical} =0.8322[/tex]

From the standard normal tables,

the p - value at [tex]Z_{critical} =0.8322[/tex] =  0.7973

Since the test is two tailed

[tex]1 - \alpha/2= 0.7973[/tex]

[tex]\alpha/2= 1-0.7973[/tex]

[tex]\alpha/2= 0.2027[/tex]

[tex]\alpha= 0.2027 \times 2[/tex]

[tex]\alpha= 0.4054[/tex]

the level of significance = 0.4054

Confidence level = 1 - level of significance

Confidence level = 1 - 0.4054

Confidence level = 0.5946

Confidence level  = 59.46%

Explain why the equation x=x+1 is a contradiction

Answers

Answer:

It results in no solution.

Step-by-step explanation:

If you subtract x on both sides, this will leave you with 0 ≠ 3. The result is no solution. This is why it is contradictory.

In the multiplication below, each of A, B and
C represents a different digit. What is ABC?
A B C
X
3
В В В

Answers

Answer:

ABC = 148, 3*148 = 444

Step-by-step explanation:

We know that 111 = 3*  37, so all numbers of the form BBB has the factor 37.

So we need a multiple of 37 such thant when multiplied, we get three digits the same as the middle digit.

Try 4*37 = 148, 148*3 = 444, bingo, we got the right combination.

So ABC is 148.

Please help me with this question

Answers

Answer:

  0 ≤ x ≤ 10

Step-by-step explanation:

The domain of f(x) is the set of values of x for which the function is defined. Here, the square root function is only defined for non-negative arguments, so we require ...

  -x^2 +10x ≥ 0

  x(10 -x) ≥ 0

The two factors in this product will both be positive only for values ...

  0 ≤ x ≤ 10 . . . . the domain of f(x)

4. Katy has 6 times as many nickels as
Shaun. Shaun has 18 nickels. How many
nickels, n, does Katy have?
n is 6
18.
n=​

Answers

Answer:

[tex]\huge\boxed{n = 108\ nickels}[/tex]

Step-by-step explanation:

Let the nickels with Katy be n

So, the condition is

n = 6 (Shaun nickels)

While Nickels of Shaun = 18 , So

n = 6 (18)

n = 108 nickels

Find the area of the shape shown below.

Answers

Answer:

28 units²

Step-by-step explanation:

Area of trapezoid =

2(8 + 4)/2 = 12

Area of rectangle =

2 x 8 = 16

16 + 12 = 28

If my answer is incorrect, pls correct me!

If you like my answer and explanation, mark me as brainliest!

-Chetan K

Open the graphing tool. Move the slider for the equation y = kx3 to a position of your choice, where k ≠ 1. Next, move the slider of y = (kx)3 so the two graphs lie on top of one another. How do the values of k compare with one another in this situation? Why do you think that is?

Answers

Answer:

For the functions to coincide, the value of k in y = (kx)3 must be smaller than in y = kx3. This is because the value of y changes more rapidly when k is cubed inside the parentheses. The behavior of the functions is similar since a vertical stretch is similar to a horizontal compression.

Step-by-step explanation:

PLATO

solve 3/4(2/3m)=24. use the properties of equality first. justify each step.

pls help urgent​

Answers

Answer:

m = 48

Step-by-step explanation:

3/4(2/3m)=24

Multiply each side by 4/3 using the multiplication property of equality

4/3*3/4(2/3m)=24*4/3

2/3m = 32

Multiply each side by 3/2 using the multiplication property of equality

3/2*2/3m = 32*3/2

m = 48

A boat takes 3 days to travel from town A to town B, but it takes 4 days to travel from town B to town A. If a motor-less raft is left alone in the water by town A, how long will it take for the raft to float to town B?

Answers

Answer:

24 days

Step-by-step explanation:

The distance from A to B equals the distance from B to A.

Let the distance between A and B be d.

3 days = 72 hours

4 days = 96 hours

speed = distance/time

speeds are in miles per hour

speed from A to B = d/72

speed from B to A = d/96

difference in speeds:

d/72 - d/96 = d/288

The speed of the water is half of the difference.

speed = d/576

When the raft floats from A to B, it uses only the speed of the water.

d/576 / d/72 = 1/8

The speed of the water is 1/8 the overall speed of the trip from A to B, so traveling by the speed of the water alone must take 8 times longer than with the boat motor.

8 * 3 days = 24 days

If 2( a^2 +b^2 ) = ( a+b)^2 , then
a. a+b =0
b. a =b
c. 2a =b
d. ab =0

Answers

Answer:

the answer is a=b

Step-by-step explanation:

What is the nearest 100 of 1730

Answers

Answer:

1700

Step-by-step explanation:

pls thnx and mark me brainliest

The cost, C, in United States Dollars ($), of cleaning up x percent of an oil spill along the Gulf Coast of the United States increases tremendously as x approaches 100. One equation for determining the cost (in millions $) is:

Answers

Complete Question

On the uploaded image is a similar question that will explain the given question

Answer:

The value of k is  [tex]k = 214285.7[/tex]

The percentage  of the oil that will be cleaned is [tex]x = 80.77\%[/tex]

Step-by-step explanation:

From the question we are told that

   The  cost of cleaning up the spillage is  [tex]C = \frac{ k x }{100 - x }[/tex]  [tex]x \le x \le 100[/tex]

     The  cost of cleaning x =  70% of the oil is  [tex]C = \$500,000[/tex]

   

Now at  [tex]C = \$500,000[/tex] we have  

       [tex]\$ 500000 = \frac{ k * 70 }{100 - 70 }[/tex]

       [tex]\$ 500000 = \frac{ k * 70 }{30 }[/tex]

      [tex]\$ 500000 = \frac{ k * 70 }{30 }[/tex]

      [tex]k = 214285.7[/tex]

Now  When  [tex]C = \$900,000[/tex]

       [tex]x = 80.77\%[/tex]

       

 

Finding Slope On a coordinate plane, a line goes through points (0, 1) and (4, 2). What is the slope of the line? m =

Answers

Answer:

slope = [tex]\frac{1}{4}[/tex]

Step-by-step explanation:

Calculate the slope m using the slope formula

m = [tex]\frac{y_{2}-y_{1} }{x_{2}-x_{1} }[/tex]

with (x₁, y₁ ) = (0, 1) and (x₂, y₂ ) = (4, 2)

m = [tex]\frac{2-1}{4-0}[/tex] = [tex]\frac{1}{4}[/tex]

Answer:

the answer would be 1/4

Step-by-step explanation:

Customers arrive at a rate of 24 people per hour to a bank. Assume that the number of customers arriving can be described using the Poisson distribution. What is the probability that at most 30 customers arrive in the next hour

Answers

Answer:

0.90415

Step-by-step explanation:

Given the following :

Arrival rate = mean(μ) = 24

Probability that at most 30 customers arrive in the next hour:

The poisson distribution formula :

P(x, μ) = [(e^-μ) * (μ^x)] / x!

Where :

e = euler's constant

P(x ⩽ 30) = p(0) + p(1) + p(2) +.... + p(30)

Using the online poisson probability distribution calculator :

P(x ⩽ 30, 24) = 0.90415

Therefore there is about 90.4% probability that at most 30 customers will arrive in the next hour.

5 x 5 = 10 x 5= 20 x 5 = Answer these 3 problems and then tell how they are
related.

Answers

Step-by-step explanation:

First we need to find out what they all 3 equal, with multiplication.

5×5=25

10×5=50

20×5=100

In each of these problems, the answer is multiplying itself by 2 in order to get to the next answer. So this is how they are related

Leena is arranging 3 different books in a row on a shelf. Create a sample space for the arrangement of a detective story (D), a mystery story (M), and a comic book (C).

Source
StylesNormal

Answers

There are 6 DIFFERENT POSSIBLE arrangements as obtained using the factorial method and they are :

DMC, CMD, DCM, MDC, CDM, MCD

Given :

DETECTIVE STORY = D

MYSTERY STORY = M

COMIC BOOK = C

To use the factorial method, we find the factorial value of the number of books to be arranged

Number of different possible arrangements = (number of books)!

Number of books = 3

Hence, 3! = 3 * 2 * 1 = 6 ways

The sample space :

DMC

CMD

DCM

MDC

CDM

MCD

THEREFORE, there are 6 different samples on the SAMPLE SPACE.

learn more :

https://brainly.com/question/13041664

Other Questions
I need 51-55 Thanks You :D no A spherical balloon has a radius of 6.95m and is filled with helium. The density of helium is 0.179 kg/m3, and the density of air is 1.29 kg/m3. The skin and structure of the balloon has a mass of 960kg . Neglect the buoyant force on the cargo volume itself. Determine the largest mass of cargo the balloon can lift. Jonathan's parents told him that for every 2 hours of homework or reading he completes, he would be able to play 30 minutes of video games. His friend Lucas'sparents told their son that he could play 1.5 hours for 5 hours of homework or reading time he completes. What is the unit rate for each boy? You must label both unit rates. PLEASE help me solve this question! No nonsense answers, and attach full solutions please! what is the volume of a unit cube? The Talbot Company uses electrical assemblies to produce an array of small appliances. One of its high cost / high volume assemblies, the XO-01, has an estimated annual demand of 8,000 units. Talbot estimates the cost to place an order is $50, and the holding cost for each assembly is $20 per year. The company operates 250 days per year. What is the economic order quantity for the XO-01 The roles and responsibilities for healthcare workers and that of their department or unit can be located? One of the main results of the French and Indian War wasA) the British and their American colonies developed a much friendlier relationship.B) several Indian tribes were allowed to settle along the Atlantic coast as a reward for theirloyalty.C) the British government expected the colonists to help pay for the huge cost of the war.D) France remained the most influential nation in North America. Which of the following describes the Mycenaean civilization? Multiple Choice burial of rulers in giant beehives construction of gigantic fortifications on the Greek mainland war with Troy that forms the setting for both the Iliad and the Odyssey All these answers are correct. 9. Light is used in photosystem II and photosystem I toO A. power the Calvin cycle.B. fix carbon dioxide.C. make carbon dioxide.D. excite an electron. The GoT cups are a fast seller and you need to ensure that you have enough rolls of paper to fulfill demand. The first stage in the process is to determine the total cost of the current inventory ordering model. Given the following information, how many rolls should they order to minimize costs?H: $1.75 per unitD: 500 rolls per monthQ: 100 units ordered at a timeS: $25 per order Hi! Which statement describes an important event in the early history ofJudaism? The Ming dynasty is known for its _____. Anne can make 6 headbands from 3/5 meters of cloth. Which expression shows the amount of cloth needed to make 1 headband? What is the solution to the expression? The expression that shows the amount of cloth needed to make 1 headband is . The solution to the expression is meters. * Distinguish between Accounts Receivable andAccount Payable. On what basis had the federal bureaucracy been established? Which TWO events happened at the same time? plsssssssssssssssssssssss help brainlist if you get my question right please I beg youuuuu. you will get 14 ppints Justify the inclusion of spirogyra in Division Algae. Q-The general solution of inequality cos 2 x- sin x is Each child in a certain class is required to have school supplies of 1 notebook and 2 pencils. One notebook costs $1.09 and one pencil costs $0.59. With $15, what is the maximum number of children that can be provided with the required supplies? (Assume no tax.) Will mark Brainlist